¿Cómo las cargas en movimiento producen campos magnéticos?

Estoy dando clases particulares a estudiantes de secundaria. Siempre les he enseñado que:

Una partícula cargada que se mueve sin aceleración produce un campo eléctrico y magnético .

Produce un campo eléctrico porque es una partícula cargada. Pero cuando está en reposo, no produce campo magnético. De repente, cuando comienza a moverse, comienza a producir un campo magnético. ¿Por qué? ¿Qué le sucede cuando comienza a moverse? ¿Qué hace que produzca un campo magnético cuando comienza a moverse?

nada le sucede a la partícula para que produzca un campo magnético cuando comienza a moverse: el campo eléctrico y magnético son componentes del campo electromagnético, que es una sola entidad, similar a cómo la energía y el momento son componentes del 4-momentum; en el marco de reposo de una partícula cargada, los componentes magnéticos se desvanecen, al igual que su 3-momentum, y solo permanecen los similares al tiempo (el campo eléctrico y la energía, respectivamente).
@Christoph: usaste muchas palabras nuevas que no entiendo. No me especializo en física. ¿Podría sugerir algo (simple) para leer?
necesitará leer sobre relatividad especial; si espera un poco, ampliaré mi comentario en una respuesta adecuada ...
Vídeo relacionado de Veritasium/Minutephysics: youtube.com/watch?v=1TKSfAkWWN0
Recomiendo encarecidamente el trabajo del profesor Oleg D. Jefimenko titulado: "Causality, Electromagnetic Induction, and Gravitation: A Different Approach to the Theory of Electromagnetic and Gravitational Fields, 2nd ed". En este trabajo, el buen profesor resuelve las ecuaciones de Maxwell y muestra que la corriente eléctrica y los campos magnéticos no son fuentes causales entre sí, sino que son una entidad dual causada simultáneamente por una fuente común.
Un imán en movimiento también tendrá un campo eléctrico. Si aceleras una partícula cargada o un imán, producirás una onda electromagnética, ya que los dos campos son una manifestación de la misma fuerza.
¿Por qué sin aceleración? Si se mueve con aceleración, ¿qué sucede?

Respuestas (7)

Si no está familiarizado con la relatividad especial, no hay forma de explicar verdaderamente este fenómeno. Lo mejor que se podría hacer es darte reglas impregnadas de ideas esotéricas como "campo electromagnético" e "invariancia de Lorentz". Por supuesto, esto no es lo que buscas, y con razón, ya que la física nunca debería tratar de aceptar reglas dictadas desde lo alto sin justificación.

El hecho es que el magnetismo no es más que electrostática combinada con relatividad especial . Desafortunadamente, no encontrará muchos libros que expliquen esto: o los autores creen erróneamente que las ecuaciones de Maxwell no tienen justificación y deben aceptarse con fe, o están demasiado sumidos en su propia notación esotérica como para detenerse a considerar qué es lo que están diciendo. El único libro que conozco que trata el tema correctamente es Electricity and Magnetism de Purcell , que recientemente se volvió a publicar en una tercera edición . (La segunda edición funciona bien si puede encontrar una copia).

Un breve esbozo heurístico de la idea es el siguiente. Supongamos que hay una línea de cargas positivas moviéndose a lo largo del z -eje en la dirección positiva - una corriente. Considere una carga positiva q situado en ( X , y , z ) = ( 1 , 0 , 0 ) , moviéndose en negativo z -dirección. Podemos ver que habrá algo de fuerza electrostática en q debido a todos esos cargos.

Pero intentemos algo loco, entremos en q marco de referencia de . Después de todo, es mejor que las leyes de la física se cumplan para todos los puntos de vista. Claramente, las cargas que constituyen la corriente se moverán más rápido en este marco. Pero eso no hace mucho, ya que después de todo, la fuerza de Coulomb claramente no se preocupa por la velocidad de las cargas, solo por su separación. Pero la relatividad especial nos dice algo más. Dice que los cargos actuales aparecerán más juntos. Si estuvieran separados por intervalos Δ z en el marco original, luego en este nuevo marco tendrán un espaciado Δ z 1 v 2 / C 2 , dónde v es q La velocidad de en el cuadro original. Esta es la famosa contracción de longitud predicha por la relatividad especial.

Si los cargos actuales aparecen más juntos, entonces claramente q sentirá una mayor fuerza electrostática del z -eje en su conjunto. Experimentará una fuerza adicional en el sentido positivo. X -dirección, lejos del eje, más allá de lo que hubiéramos predicho simplemente sentados en el marco del laboratorio. Básicamente, la ley de Coulomb es la única ley de fuerza que actúa sobre una carga, pero solo el marco de reposo de la carga es válido para usar esta ley para determinar qué fuerza siente la carga.

En lugar de transformarnos constantemente de un lado a otro entre cuadros, inventamos el campo magnético como un dispositivo matemático que logra lo mismo. Si se define correctamente, explicará por completo esta fuerza anómala que aparentemente experimenta la carga cuando no la observamos en su propio marco de reposo. En el ejemplo que acabo de ver, la regla de la mano derecha le dice que debemos atribuir un campo magnético a la corriente que circula alrededor del z -eje tal que está apuntando en el positivo y -dirección en el lugar de q . La velocidad de la carga es negativa. z -dirección, y así q v × B puntos en positivo X -dirección, tal como aprendimos al cambiar los marcos de referencia.

Muchas gracias por decirme lo que no sé en lugar de responder directamente a mi pregunta. Gracias por la recomendación del libro. Gracias por usar un lenguaje tan simple para explicar cosas nuevas para mí. Me encantó tu respuesta :)
¡Guau! (+1) Una de las legendarias malditas buenas respuestas a una pregunta aparentemente simple, que vemos aquí de vez en cuando. Me recuerda a eso de la energía cinética y los pedazos de arcilla de RM
No creo que esa sea toda la historia. El campo electromagnético sólo puede reducirse a uno eléctrico en caso de PAGS < 0 . En caso de PAGS > 0 , su descripción se descompone y se vería obligado a considerar la magnetostática como la interacción fundamental que se impulsa a diferentes marcos...
@Christoph No estoy tratando de deshacerme del campo magnético, ya que eso implicaría transformarse en el marco de reposo de las cargas que lo producen, y claramente ese marco global no existirá para la mayoría de las distribuciones actuales. Me estoy transformando en el marco de la carga de prueba , que siempre se puede hacer y en la que no hay efecto del magnetismo sobre la carga.
@ChrisWhite: gracias por la aclaración, no pude entender la premisa de su configuración; Sin embargo, un campo magnético que no se desvanece generalmente tendrá un efecto en la carga de prueba (alineación del momento magnético intrínseco); ¿Se puede modelar eso también con sensatez, por ejemplo, haciendo girar el marco de reposo de la carga de prueba?
Giro intrínseco de @Christoph: ahora eso es una complicación. Admito que no lo he pensado todo.
Buen intento, pero realmente deberías haber copiado el argumento de Purcell donde considera que una corriente de electrones se mueve en un cable donde los campos eléctricos se cancelan entre sí en el marco del laboratorio, pero no en el marco del electrón.
@ChrisWhite Tenía una pregunta sobre su respuesta: ¿no debería permanecer constante el espacio entre la carga actual? Pregunto eso porque, de la paradoja de la nave espacial de Bell, mi impresión es que para un observador 'estacionario' que ve dos cuerpos separados por un espacio 'x' acelerar a una velocidad 'v', entonces no observaría una diferencia en la brecha entre los cuerpos, aunque ellos mismos se contraerían.
@ user1218748 En realidad, nunca antes había oído hablar de esa paradoja. Parece depender de las sutilezas de cómo se implementa la aceleración (específicamente cuando se detiene). Los marcos acelerados son complicados, especialmente cuando se trata de extenderlos a largas distancias. Ciertamente, debe tener cuidado, porque su interpretación significaría que la contracción de longitud no puede ocurrir en absoluto, solo vea todos los objetos como colecciones de sus átomos. En cualquier caso, esto suena como la base para una pregunta separada.
@ChrisWhite: suponga que su carga se acerca a la corriente en lugar de moverse en paralelo a ella. ¿Qué predices entonces? Hay más en E&M que Coulomb + SR. Revise Jackson 12.2.
Para su información, Griffiths también le da un buen tratamiento a esto en su libro.
Quizás estoy tomando sus palabras demasiado literalmente, pero creo que la frase "El magnetismo no es más que electrostática combinada con SR" es falsa. El ejemplo más fácil que no puede tratar con electrostática es el de una carga acelerada. Como enseña Feynman en la lección 26, puede obtener los campos completos de la ley de Couloumb si agrega las ideas de que (1) el potencial ( ρ , A X , A y , A z ) es un tetra-vector y (2) que solo depende de posiciones y velocidades en el tiempo retardado.
¿Existe un análogo para las otras fuerzas, por ejemplo: la gravedad? No veo ninguna razón por la que no debería haberlo, pero al mismo tiempo no estoy al tanto de nada parecido.
Me temo que esta respuesta es incorrecta. Se repite un mito. No puede derivar el movimiento magnético rotacional de la contracción de la longitud en un cable lineal. Vea esta pregunta y la discusión asociada.
@JohnDuffield Sobre la pregunta que vinculó, me parece que la respuesta más votada está de acuerdo con esta respuesta, ¿o leí mal algo?
180 votos, pero está mal y le pido que le dé una edición importante a su respuesta. La cuestión de si el magnetismo puede derivarse completamente de la electrostática más la relatividad se ha considerado durante mucho tiempo y ciertamente no es posible. Una razón es que no le dirá los efectos de la aceleración. También, B no solo contribuye con una fuerza sobre una partícula en movimiento, también actúa como una fuente para mi por inducción ¿Cómo se explica una onda electromagnética de la electrostática? No se puede hacer. Finalmente, siempre que mi B 0 no hay marco en el que B desaparece
Soy profano, pero esta respuesta me molesta por dos cosas: - La velocidad de deriva de un electrón a través de un alambre de cobre de sección transversal 3,00 x 10-6 m², con una corriente de 10 A será de aproximadamente 2,5 x 10 -4 m/s, una velocidad relativista muy baja. El campo eléctrico estará en la velocidad de la luz dura. - También se puede producir un campo eléctrico moviendo un imán. Incluso puedes producir una onda electromagnética aceleras el imán, el mismo comportamiento que tiene una partícula eléctrica. ¿No se responde mejor a estas preguntas con la mecánica cuántica y su teoría de campos?
Solo quiero señalar que no está claro por qué qse requiere una carga de prueba para moverse. La explicación tiene sentido (ya sea incorrecta o correcta) cuando la carga de prueba no se mueve, a menos que me esté perdiendo algo.
¿Por qué esa explicación no predice el "campo magnético gravitatorio"? Reemplace las cargas con masas puntuales y la repulsión de Coulomb con atracción gravitatoria; ¿No verías el mismo efecto? De hecho, el efecto de las masas más próximas entre sí se vería agravado por el aumento relativista de las masas individuales.
Desafortunadamente, esta respuesta es incorrecta. La idea de que el campo magnético es simplemente el efecto relativista del campo eléctrico ha sido considerada hace décadas; Creo que Feynman lo mencionó en alguna parte con el propósito de explicar por qué está mal . Las transformaciones de Lorentz explican por qué los efectos del campo electromagnético son consistentes independientemente del marco de coordenadas. También parecería que son ejemplos de juguetes que la electrostática + Lorentz es todo lo que se necesita para explicar las cosas, pero no, en general, esa no es una imagen completa.

Los campos eléctricos y magnéticos son lo que el campo electromagnético ' parece ' desde un marco de referencia particular (inercial).

Tome una partícula cargada: en su marco de reposo, parece generar solo un campo eléctrico y ningún campo magnético. Desde un marco de referencia diferente (en particular, uno en movimiento relativo), veremos la carga en movimiento, por lo tanto, una corriente que también genera un campo magnético.

Esto no significa que poner la partícula en movimiento de alguna manera accionó un interruptor dentro de la partícula; más bien, es un artefacto de nuestra elección de marco de referencia: los observadores en movimiento relativo medirán diferentes intensidades de campos eléctricos y magnéticos de la misma manera que miden diferentes. velocidades y momentos.

Sin embargo, existen invariantes del campo electromagnético, es decir, cosas en las que todos los observadores pueden estar de acuerdo y, en particular,

PAGS = B 2 mi 2 q = mi B

Tomemos un campo em distinto de cero con PAGS , q = 0 , es decir mi 2 = B 2 y mi B . Un ejemplo sería una onda electromagnética plana, que parecerá una onda plana para todos.

Ahora deja PAGS 0 pero q = 0 . Entonces, podemos encontrar marcos de referencia donde ya sea el eléctrico (en caso de PAGS > 0 ) o el campo magnético (en caso de PAGS < 0 ) desaparece. El marco de reposo de nuestra partícula cargada sería tal.

Para obtener más detalles, deberá buscar en la literatura sobre relatividad especial.

Entonces, ¿eso significa que una partícula cargada no produce un campo eléctrico en movimiento?
¿Cómo podemos restar B 2 de mi 2 cuando tienen diferentes unidades?
@ Asher2211, las unidades que se utilizan aquí son unidades cgs, que no son infrecuentes para el electromagnetismo teórico. en Unidades SI el invariante es B 2 mi 2 C 2

Aunque la respuesta de Chris White a la pregunta "¿Por qué las cargas en movimiento producen un campo magnético?" publicado por un profesor de secundaria (Claws) el año pasado, fue seleccionado como la mejor respuesta, creo que contiene varias trampas. Chris White imagina una corriente de cargas positivas fluyendo en el + z dirección del eje, mientras que una carga de prueba + q ubicado inicialmente en ( 1 , 0 , 0 ) se mueve en sentido contrario ( z ) dirección con velocidad v . A continuación, intenta demostrar que cuando el observador se ubica en el marco de la carga de prueba en movimiento, verá, además de la fuerza electrostática regular de Coulomb (repulsión) que actúa sobre la carga de prueba, una repulsión adicional en el + X dirección cuyo origen es enteramente relativista. Esto sucede, dice, porque la separación original Δ z 0 entre los cargos (cuando se ve desde el marco de descanso del laboratorio) ahora se contrae para Δ z = Δ z 0 ( 1 v 2 / C 2 ) (La “famosa” contracción de Lorentz).

En consecuencia, todas las distancias de las cargas que fluyen a la carga de prueba se vuelven más pequeñas (como si la densidad de carga aumentara) y, por lo tanto, las repulsiones de Coulomb también aumentan. Este exceso de repulsión es la fuerza magnética “ilusoria” que ve el observador del laboratorio cuando la carga de prueba se mueve en el z dirección con velocidad v .

En resumen: no hay fuerza magnética intrínseca. Todo es fuerza de Coulomb, vista desde el marco de laboratorio (fuerza electrostática pura), o vista desde el marco de carga en movimiento (electrostática más repulsión de Coulomb). Podemos pasar por alto aquí todos los detalles cuantitativos que White también omite, pero no podemos pasar por alto las trampas:

  1. Primero hay una contradicción verbal: notar lo contraído Δ z , menor que Δ z 0 , el observador debe ubicarse en reposo con la carga q (es decir, moviéndose con la carga). Pero luego, al final, White dice que la nueva “fuerza anómala aparentemente experimentada por la carga” (es decir, el campo magnético definido), ocurre “cuando no la estamos observando en su propio marco de reposo” (énfasis mío). Entonces, ¿cuál es el trato? Para predecir la fuerza extra de Coulomb (magnética) tenemos que adoptar el marco de la carga en movimiento. Pero para observarlo tenemos que permanecer en el marco Lab, que NO es el marco de carga en movimiento.
  2. Del mismo modo, existe un escollo numérico: la nueva separación de carga (contraída) Δz observada desde el marco de la carga en movimiento se calcula como Δ z = Δ z 0 ( 1 v 2 / C 2 ) dónde v , dice White, es “ q La velocidad de en el cuadro original”. No debería haber puesto v pero 2 v , ya que la velocidad relativa entre la corriente de carga que sube, v , y la carga de prueba bajando, v , es v ( v ) = 2 v . Entonces el factor de contracción debe ser 1 4 v 2 / C 2 .
  3. Además, si usamos la estrategia heurística utilizada por White, llegamos a una contradicción: Empezar con todas las cargas en reposo: la z eje lleno de cargas y la carga de prueba en ( 1 , 0 , 0 ) . Llamar Δ z 0 la separación entre todas las cargas en reposo. Ahora permita el z El eje se carga para moverse como antes, con una velocidad + v . Ya el observador de laboratorio Y LA CARGA DE PRUEBA q , verá una contracción de la separación según Δ z = Δ z 0 ( 1 v 2 / C 2 ) . Por lo tanto, mediante las mismas maniobras que antes, el pariente especial debe predecir una repulsión de "Coulomb" adicional debido a la densidad de carga compactada. Entonces, la fuerza "magnética", así predicha, debe actuar sobre la carga en REPOSO en ( 1 , 0 , 0 ) . Y esto no se observa. Que yo sepa, no hay corriente a lo largo del z El eje puede producir una fuerza magnética sobre una carga en reposo en el origen.

En conclusión: contrariamente a lo que dice White, el magnetismo NO es SOLO electrostática más relatividad especial. Tal visión reduccionista convierte el magnetismo en un juego superficial entre marcos de referencia.

Nick Stauner: gracias por la edición, pero las ecuaciones relativistas están todas mal. Usé superíndices en el original para v^2 y c^2 y también para la raíz cuadrada usé ^1/2 pero ahora las potencias aparecen como subíndices y la raíz cuadrada se convirtió en la fracción 1/2. Por favor, vea que las ecuaciones se restauran. Gracias
Gracias Kyle, por reconstruir mis ecuaciones originales.
Por lo que puedo decir, esta no es una crítica válida a la respuesta de White. El único punto válido es 2., donde de hecho cometió un error numérico, pero esto no cambia el resultado cualitativo de su explicación (y, por cierto, su versión sigue siendo errónea). Todo lo que White dice es que todos los efectos magnéticos observados en cualquier marco de inercia pueden explicarse solo por las fuerzas de Coulomb en el marco de reposo de q , y no veo ningún punto en que usted contradiga esto. Especialmente la "fuerza adicional" que está describiendo en 3. actuando sobre q no hay fuerza adicional en q Marco de descanso. Sólo cuando lo dejamos.

La carga produce un campo que actúa sobre otras cargas. Pero la acción de este campo se ve diferente desde diferentes marcos de referencia.

Por definición,

  • campo eléctrico es algo que acelera otras cargas, y
  • El campo magnético es algo que hace girar otras cargas.

Considere la carga en reposo. Produce solo campo eléctrico en su marco de reposo. En este marco actúa sobre otras cargas acelerándolas en la dirección del campo eléctrico. mi . Lo que vemos en el marco de reposo de la carga es que los vectores de momento de otras cargas en este marco están "impulsados".

Sin embargo, si observamos esto desde el marco en movimiento, veremos que los vectores de impulso de otras cargas no solo se " aceleran ", sino que también se " rotan ".

Esto se debe simplemente a que la aceleración "pura" en un cuadro parece una combinación de aceleración y rotación en otro cuadro.

Para dar cuenta de este "nuevo efecto" - la rotación del vector de momento - los físicos dicen que en el segundo marco (que se mueve con la carga) hay un campo magnético (además del campo eléctrico que (por definición, ver arriba) solo acelera otras cargas).

-1 Un campo magnético también acelera las cargas.
La rotación es aceleración, ¿no?
si, eso es correcto

Una "primera respuesta" simple sería usar la analogía de un bote en un lago. Cuando el bote se mueve sobre la superficie del agua, perturba el agua y crea ondas. Cuando no se mueve, no lo hace.

De manera similar, cuando una partícula cargada se mueve a través del campo EM (espacio), perturba el campo EM y genera un campo magnético perpendicular a la dirección del movimiento de la partícula.

Luego puede usar cualquiera o todas las otras respuestas que haya recibido para entrar en más detalles.

Excelente respuesta los mi -field se puede considerar como un tipo de "fluido" que llena el espacio. Una partícula cargada en movimiento a través de este fluido crea ondas perpendiculares que pueden interpretarse como la B -campo. Están vinculados: no se puede tener uno sin el otro.
@Guill, lo siento por la pregunta tonta, pero ¿el campo magnético es simplemente el movimiento / onda (causado por el movimiento de la carga) en el espacio (que está hecho del campo eléctrico que es como un fluido)?

Suponga que tiene dos cargas. Uno está en el origen de nuestro sistema de coordenadas. El otro está en alguna posición arbitraria. ( X , y , z ) y supongamos que alguna fuerza mágica lo mantiene allí, independientemente de los campos EM que puedan ocurrir allí.

Suponga que la carga en el origen se mueve en línea recta a velocidad constante. La carga objetivo solo recibe actualizaciones de la ubicación de la carga en movimiento a la velocidad de la luz. Responderá a la carga en movimiento, no según dónde se encuentre ahora, sino dónde estuvo en el pasado.

A medida que la carga en movimiento se acerca a la carga objetivo, parte del efecto cancelará el efecto debido a la carga anterior en su trayectoria. Lo contrario sucede cuando la carga se aleja. Habrá algo de cancelación del campo debido a los efectos de superposición con esta cancelación ocurriendo en el componente paralelo a la dirección del movimiento.

Una carga en movimiento incide sobre un objetivo desde una distancia diferente a lo largo del tiempo. Una carga en movimiento incide sobre un objetivo desde una dirección diferente a lo largo del tiempo. Los efectos cambiantes tienen un retraso antes de llegar al objetivo.

Me gusta tu explicación, ya que no requiere dos sistemas de coordenadas separados. ¿Existen explicaciones detalladas sobre cómo cambia la interacción entre las cargas a medida que se mueve una carga? Sería bueno tener alguna aproximación discreta.
@user1700890: en.wikipedia.org/wiki/Li%C3%A9nard%E2%80%93Wiechert_potential Es un potencial eléctrico para una carga en movimiento que tiene en cuenta los efectos relativistas. El potencial magnético se toma multiplicándolo por la velocidad y luego dividiéndolo por el cuadrado de la velocidad de la luz. Hay varios "factores de corrección" que aparecen en las expresiones de los campos que lo diferencian del caso estático.

Es posible que desee decir "El campo eléctrico de una carga en reposo aparece como un campo eléctrico y un campo magnético cuando se ve desde un marco de referencia en movimiento". Los comentarios lo hacen bien, una carga está asociada con un campo electromagnético. Aparece como un campo electrostático cuando se ve desde un marco en el que la carga está en reposo.